Proving a property of the dimension of eigenspaces in a finite dimensional space

Click For Summary
If A is a linear map on a finite-dimensional space V with dimension n, and the geometric multiplicities of k distinct eigenvalues sum to n, then A cannot have any additional eigenvalues. The argument is based on the fact that eigenvectors corresponding to distinct eigenvalues are independent. If a new eigenvalue existed, it would introduce an additional independent eigenvector, resulting in n+1 independent vectors, which contradicts the dimension of V. Therefore, the assumption of an additional eigenvalue must be false. This conclusion reinforces the relationship between eigenvalues, eigenvectors, and the dimensionality of the space.
cloverforce
Messages
3
Reaction score
0

Homework Statement



Prove that if A: V - >V is a linear map, dim V = n, and h1,...,hk (where 1,...,k are subscripts) are pairwise different eigenvalues of A such that their geometric multiplicities sum to n, then A does not have any other eigenvalues.

Homework Equations


Note sure if this is relevant or not, but if hj and hk are distinct eigenvalues, then the intersection of V(hj) and V(hk) is {0}.

The Attempt at a Solution


My attempt has been by contradiction. Suppose that there exists an eigenvalue h distinct from the k terms already given. In that case, I want to show that dim(V(h)) = 0, which would mean that its basis is the empty set and thus no such eigenvalue can exist. I'm not sure why this would have to be the case, though.
 
Last edited:
Physics news on Phys.org
One important property you will need is that eigenvectors corresponding to distinct eigenvalues are independent.

If "their geometric multiplicities sum to n" then, by definition of "geometric multiplicity" you already have n independent vectors. If there existed another eigenvalue, distinct from any of the previous ones, then an eigenvector corresponding to it would be independent of the first n, giving you n+1 independent eigenvectors.
 
Question: A clock's minute hand has length 4 and its hour hand has length 3. What is the distance between the tips at the moment when it is increasing most rapidly?(Putnam Exam Question) Answer: Making assumption that both the hands moves at constant angular velocities, the answer is ## \sqrt{7} .## But don't you think this assumption is somewhat doubtful and wrong?

Similar threads

Replies
34
Views
3K
  • · Replies 1 ·
Replies
1
Views
1K
  • · Replies 15 ·
Replies
15
Views
2K
Replies
7
Views
2K
  • · Replies 13 ·
Replies
13
Views
2K
  • · Replies 2 ·
Replies
2
Views
3K
  • · Replies 4 ·
Replies
4
Views
3K
  • · Replies 14 ·
Replies
14
Views
3K
  • · Replies 3 ·
Replies
3
Views
2K
  • · Replies 4 ·
Replies
4
Views
2K